LSAT and Law School Admissions Forum

Get expert LSAT preparation and law school admissions advice from PowerScore Test Preparation.

 Administrator
PowerScore Staff
  • PowerScore Staff
  • Posts: 8927
  • Joined: Feb 02, 2011
|
#26546
Complete Question Explanation
(The complete setup for this game can be found here: lsat/viewtopic.php?t=11086)

The correct answer choice is (D)

If M is faster than J, then at least three chips must be faster than J: H, M, and F or G. Thus, the earliest J could be is fourth, eliminating answer choices (A) and (B). However, in order to be certain that J can be fourth, try a hypothetical and confirm that J can be fourth:
June 06_game#4_M12_L3_explanations_game#3_#21_diagram_1.png
Unfortunately, this hypothetical fails because the first four spaces are occupied, forcing K to be ranked fifth and thus L must be ranked eighth, leaving no room for O. Therefore, answer choice (C) is also incorrect.

J in fifth will work, as shown by the following hypothetical:
June 06_game#4_M12_L3_explanations_game#3_#21_diagram_2.png
Answer choice (D) is therefore correct.
 andbzav@gmail.com
  • Posts: 17
  • Joined: Jul 18, 2019
|
#66773
Could it be said that the H_J block (with 2 spaces before J) + M (a third space before J), along with the fact that F or G has to come before H (a fourth space before J), forces the same outcome - that the fastest ranking J could be is 5th?
 Adam Tyson
PowerScore Staff
  • PowerScore Staff
  • Posts: 5153
  • Joined: Apr 14, 2011
|
#66775
You could get to it that way IF you made the inference that M cannot be the variable that is directly between H and J, which many students unfortunately miss. Since that space must be either filled by K or L, you could say that J must now be after H, M, either K or L, and either F or G.

If you fail to make that inference back in the setup portion of the game, you might need to do the extra work described here.
 andbzav@gmail.com
  • Posts: 17
  • Joined: Jul 18, 2019
|
#66799
Thank you for your time.
 mackenziemarcus
  • Posts: 3
  • Joined: Jan 22, 2021
|
#84032
Can you explain why there is no room for O?

I can't see the rule against the order being F G M J K O G L
 mackenziemarcus
  • Posts: 3
  • Joined: Jan 22, 2021
|
#84033
mackenziemarcus wrote: Fri Feb 12, 2021 2:08 pm Can you explain why there is no room for O?

I can't see the rule against the order being F G M J K O G L
I made a typo I mean this order: F H M J K O G L
 Robert Carroll
PowerScore Staff
  • PowerScore Staff
  • Posts: 1774
  • Joined: Dec 06, 2013
|
#84038
mackenzie,

O must be slower than J and L, via the last rule, so that order is not acceptable.

Robert Carroll
 alliewad95
  • Posts: 4
  • Joined: Feb 19, 2020
|
#84297
I'm so confused... Why is B wrong? Can you please submit a step by step answer; thank you in advance.
 Adam Tyson
PowerScore Staff
  • PowerScore Staff
  • Posts: 5153
  • Joined: Apr 14, 2011
|
#84353
We already did, alliewad95 - it's in the first post at the top of this thread. I'm not sure I could add anything to that! Give that a look and let us know if there is anything about it that you found confusing.

Also, if you are unsure about answer B, try it out! Draw a hypothetical solution where J is third. Remember, all the original rules still apply, so F or G must be first, and H is before J with exactly one chip between them. And of course, per the local restriction in this question, M has to also be somewhere before J (which could perhaps put it between the H and the J, although you will discover that's not possible because of the effect that would have on the K _ _ L block).

Give it a try and let us know if you are still having trouble with it!
User avatar
 josh1ua
  • Posts: 6
  • Joined: Jun 13, 2021
|
#88212
Why doesn't the M go between the H and J when the question says that it M is faster J? Also, what would the diagram be for this question?

Get the most out of your LSAT Prep Plus subscription.

Analyze and track your performance with our Testing and Analytics Package.